Jump to content

Uchiha sisui's Content

There have been 175 items by Uchiha sisui (Search limited from 07-06-2020)



Sort by                Order  

#697774 TOPIC Luyện tập về ứng dụng của tỉ số kép và hàng điểm điều hòa

Posted by Uchiha sisui on 04-12-2017 - 20:05 in Hình học

Bài 4. (China TST 2002) 

 

Cho tứ giác lồi $ABCD$, gọi $E, F, P$ lần lượt là giao điểm của $AD$ và $BC$, $AB$ và $CD$,  $AC$ và $BD$. Gọi $O$ là chân đường cao hạ từ $P$ xuống $EF$. Chứng minh rằng $\angle AOD=\angle BOC$ 

 

Attached Images

  • 3.png



#701510 Cho tam giác ABC vuông tại C, biết B(0;1). Đường tròn tâm (I) ngoại tiếp tam...

Posted by Uchiha sisui on 11-02-2018 - 20:40 in Phương pháp tọa độ trong mặt phẳng

Lời giải

 

  • Trước tiên ta chứng minh $EA=EI$. 

 

Thật vậy từ điểm $M$ kẻ hai tiếp tuyến $MX$, $MY$ đến $(I)$. $XY$ cắt $AB$ tại $P$. Theo hệ thức quen thuộc ta có: $\frac{PA}{PB}=\frac{MA}{MB}=\frac{1}{3}$ suy ra $P$ là trung điểm của $AI$.

 

Gọi $H$ là trực tâm tam giác $EAB$ áp dụng định lý Pascal cho bộ 6 điểm $(ACBDXY)$ suy ra $E,Y,H,X,P$ thẳng hàng.

 

Do đó $EP$  vuông góc với $AI$ suy ra tam giác $EAI$ cân hay $EA=EI$

 

  • Quay trở lại với bài toán.

 

Chú ý  rằng $MA=AI=IB$ và có tọa độ điểm $M,B$ nên ta dễ dàng tính được tọa điểm các điểm $A,I$. Lại có $EA=EI$ nên ta tính được tọa độ điểm $E$.

 

Sử dụng phương tích ta có: $EC.EA=EI^{2}-R^{2}$ ($R$ là bán kính đường tròn $(I)$) Từ đây ta tính được tọa độ điểm $C$. Tương

 

tự ta cũng tính được tọa độ điểm $D$.

 

  • Lời bình: Bài này có lẽ tôi giải hơi hâm hâm, chắc còn cách khác, chờ cao nhân! Nhưng việc trong Oxy mà vận dụng các kiến thức hình phẳng là điều khá quen thuộc trong các đề thi Đại Học gần đây! Có lẽ năm sau chờ bộ ra trắc nghiệm Oxy 

 

 

Attached Images

  • Untitled.png



#697771 TOPIC Luyện tập về ứng dụng của tỉ số kép và hàng điểm điều hòa

Posted by Uchiha sisui on 04-12-2017 - 19:53 in Hình học

Bài 3. (9) VMO 2010 

 

Cho tam giác $ABC$ nội tiếp $(O)$ cố định $B,C$  và $A$ di chuyển trên $(O)$. Gọi phân giác trong và ngoài của tam giác lần lượt là $AD$ và $AE$ với $D,E$ thuộc $BC$. $M$ là trung điểm của $DE$. $H$ là trực tâm của tam giác $ABC$. Chứng minh rằng đường thẳng qua $H$ vuông góc với $AM$ luôn đi qua một điểm cố định khi $A$ di chuyển trên $(O)$.

 

 

Attached Images

  • 2.png



#697769 TOPIC Luyện tập về ứng dụng của tỉ số kép và hàng điểm điều hòa

Posted by Uchiha sisui on 04-12-2017 - 19:42 in Hình học

Bài 2.

 

Ta có: $MB.MK=ML.MC$ suy ra $M$ thuộc trục đẳng phương của $(CFL)$ và $(KBE)$.

 

Do đó $MP$ cũng là trục đẳng phương của hai đường tròn này nên $T$ cũng thuộc đường thẳng đó. 

 

Do đó $TB.TE=TC.TF$ suy ra $T$ thuộc trục đẳng phương của $(ABE)$ và $(ACF)$ nên giao điểm khác $A$ của hai đường tròn nằm trên $AT$.

 

 

Attached Images

  • 1.png



#604656 CMR: Tồn tại các số tự nhiên a,b,c

Posted by Uchiha sisui on 22-12-2015 - 16:47 in Số học

bài này dễ mà




#701511 Cho tam giác ABC vuông tại C, biết B(0;1). Đường tròn tâm (I) ngoại tiếp tam...

Posted by Uchiha sisui on 11-02-2018 - 20:41 in Phương pháp tọa độ trong mặt phẳng

Có lẽ bài của bạn là bài này tôi đã giải ở đây. Đúng trong ngày hôm nay, tình cờ thật  :icon6:

 

https://diendantoanh...c-abc/?p=701510




#701690 Chứng minh rằng $O,I,M,N$ đồng viên.

Posted by Uchiha sisui on 15-02-2018 - 12:05 in Hình học

Bạn sử dụng bài toán này là sẽ chứng minh được bài toán của bạn thôi !  :closedeyes:  :closedeyes:

 

https://diendantoanh...rd/#entry701689

Attached Images

  • 6.png



#604717 Bài toán tổng quát trong Cân bằng hệ số

Posted by Uchiha sisui on 22-12-2015 - 20:14 in Chuyên đề toán THCS

tôi mới học lớp 8 nên không biết Nhân tử langrange




#604699 Bài toán tổng quát trong Cân bằng hệ số

Posted by Uchiha sisui on 22-12-2015 - 19:34 in Chuyên đề toán THCS

Bài bạn ngược dấu rồi.Mà đừng có bao giờ bảo đề Thái Bình dễ nha bạn  :icon6:

Cho các hằng thức dương a,b,c và các biến số x,y,z thỏa mãn điều kiện xy + yz + zx $\geq 1$ .Tìm giá trị nhỏ nhất của biểu thức P=$ax^{2} + by^{2} + cz^{2}$




#604725 Bài toán tổng quát trong Cân bằng hệ số

Posted by Uchiha sisui on 22-12-2015 - 20:27 in Chuyên đề toán THCS

Em đã coi qua tài liệu cân bằng hệ số nhưng vẫn chưa tìm được cách chứng minh bài này có gì mong diễn đàn giúp em




#657366 $a^{2}+b^{2}+c^{2}=3$

Posted by Uchiha sisui on 09-10-2016 - 22:32 in Bất đẳng thức và cực trị

Cho các số thực dương a,b,c sao cho $a^{2}+b^{2}+c^{2}=3$.Chứng minh rằng : $\frac{a}{b} + \frac{b}{c} + \frac{c}{a} \geq \frac{9}{a+b+c}$



#702901 ĐỀ KIỂM TRA LỚP CHUYÊN LẦN 3 - THPT CHUYÊN LÊ HỒNG PHONG, THÀNH PHỐ HỒ CHÍ MINH

Posted by Uchiha sisui on 05-03-2018 - 22:53 in Thi HSG cấp Tỉnh, Thành phố. Olympic 30-4. Đề thi và kiểm tra đội tuyển các cấp.

Lời giải của tôi cho bài toán này!

 

Gọi $Z$ là điểm đối xứng của $M$ qua $A$. Từ $Z$ hạ các đường vuông góc xuống $BC, CA, AB$ tại $X', T, Y$. Suy ra tứ giác $AHZX$ là hình bình hành.'

 

Để ý là $M$ là trực tâm của tam giác $APQ$ nên bằng cộng góc ta chứng minh được 6 điểm $M, P, Q, X', T, Y$ cùng nằm trên một đường tròn. 

 

Từ đây suy ra $X'\equiv X$ $\Rightarrow MH=MX\Rightarrow BH=CX$.

 

Vậy ta có điều phải chứng minh

 

Attached Images

  • Untitled.png



#691107 Chứng minh rằng $OP\perp AQ$.

Posted by Uchiha sisui on 20-08-2017 - 11:13 in Hình học

sao mình thây trong quyển tài liệu chuyên toán ko có nhỉ 

 

Muôn nơi.




#690445 Chứng minh rằng $OP\perp AQ$.

Posted by Uchiha sisui on 13-08-2017 - 18:50 in Hình học

Dựng $A'$ thuộc $SP$ sao cho $AA' || BC$, khi đó $O(AD, PM)=(AD, PM)=(A'Q, PS)=A(SP, QA')\Rightarrow OP\perp AQ$

Cho mình hỏi tại sao hai chùm điều hòa bằng nhau lại vuông góc




#690954 Chứng minh rằng $OP\perp AQ$.

Posted by Uchiha sisui on 18-08-2017 - 21:24 in Hình học

Bạn có thể cho mình hỏi định lí này trong tài liệu nào được không?

 

Mình có định lý sau: Xét hai chùm $x,y,z,t$ và $x', y', z', t'$ thỏa mãn $x\perp x', y\perp y', z\perp z'$. Khi đó $(xy, zt)=(x'y', z't')\Leftrightarrow t\perp t'$

Ở trên có $O(AD, PM)=A(SP, QA')$, có $OA\perp AS, OD\perp AP, OM\perp AA'$ nên $AP\perp AQ$

 




#685731 Đề chọn đội tuyển Anh dự thi IMO 2005

Posted by Uchiha sisui on 27-06-2017 - 14:38 in Bất đẳng thức - Cực trị

Trước đây bài đề này đã từng được thảo luận ở topic: https://diendantoanh...-3/#entry363791

 

 

Ngoài ra

\[\sum \dfrac{a+3}{(a+1)^{2}} - 3 = \frac12\sum \frac{(abc^2+3)(a-b)^2+ab(a+b+2)(c-1)^2}{(a+1)^2(b+1)^2(c+1)^2} \geqslant 0.\]

Anh Huyện còn link tải phần mềm Maple của anh  không ? Nếu có thì cho em xin link với, em vào blog cũ của anh link hỏng hết rồi!




#709353 Cho tam giác ABC. Một đường tròn bất kì qua B,C cắt AC,AB tại E,F...

Posted by Uchiha sisui on 27-05-2018 - 11:25 in Hình học

Bài này là một bổ đề rất quen thuộc với tứ giác nội tiếp rồi. Có thể dùng tỷ số kép hoặc cách dựng thêm hình bình hành 

Mình sẽ trình bày cách sử dụng tỷ số kép:

 

Bỏ qua trường hợp đơn giản tam giác $ABC$ cân tại $A$

Gọi $P$ là trung điểm của $AD$. Gọi $K,H$ lần lượt là giao điểm của $AD$ với $EF,BC$

Gọi $G$ là giao điểm của $EF$ và $BC$

Theo đường thẳng $Gauss$ thì $M,N,P$ thẳng hàng

Do đó ta cần chứng minh: $PA^2=PD^2=PN.PM$

Ta có: $(AD,KH)=-1$ nên theo hệ thức $Newton$ thì $PA^2=PK.PH$ do đó chỉ cần chứng minh $PM.PN=PH.PK$ hay $KNMH$ nội tiếp

Lại có: $(GH,BC)=-1$ và $(GK,FE)=-1$ nên theo hệ thức $Maclaurin$ thì

$GH.GM=GB.GC=GF.GE=GK.GN \Rightarrow HKNM$ nội tiếp  

Kết hợp các điều trên ta có điều phải chứng minh.

Bài này là IMO Shortlish 2009




#684608 CMR: $a^2+b^2+c^2+a+b+c\geq \frac{18}{a+b+c...

Posted by Uchiha sisui on 15-06-2017 - 11:28 in Bất đẳng thức - Cực trị

Bài này sử dụng PQR   :luoi:  đơn giản như sau: 

 

Đặt $p= a+b+c, q=ab+bc+ca, r=abc$

 

Áp dụng BĐT AM-GM ta có: $p=a+b+c\geq 3\sqrt{abc}=3\Rightarrow p-3\geq 0$ (*)

 

Sử dụng bất đẳng thức SCHUR ta có $r\geq \frac{p(4q-p^{2})}{9}$$\Rightarrow 4pq\leq p^{3}+9$

 

Bất đẳng thức cần chứng minh tương đương với: $p^{3}-2pq+p^{2}\geq 18$

 

$\Leftrightarrow 2p^{3}-4pq+2p^{2}-36\geq 0 \Leftrightarrow p^{3}+2p^{2}-45\geq 0$

 

$\Leftrightarrow (p-3)(p^{2}+5p+15)\geq 0$     (**)

 

Theo (*) thì Bất đẳng thức (**) luôn đúng. Vậy ta có điều phải chứng minh. Dấu bằng xảy ra $\Leftrightarrow a=b=c=1$

:closedeyes:  :closedeyes:  :closedeyes:

 
 



#684633 CMR: $a^2+b^2+c^2-4abc+1\geq 0$

Posted by Uchiha sisui on 15-06-2017 - 15:18 in Bất đẳng thức - Cực trị

Áp dụng bất đẳng thức $Cauchy-Swharz$,ta có:

$3=\frac{1}{ab}+\frac{1}{bc}+\frac{1}{ca}\geq \frac{9}{ab+bc+ca}\geq \frac{9}{\frac{(a+b+c)^2}{3}} \Leftrightarrow a+b+c\geq 3$.

Mặt khác biến đổi giả thiết,có:$a+b+c=3abc \Rightarrow 4abc=\frac{4}{3}(a+b+c)$.

Lúc này:$a^2+b^2+c^2-4abc+1\geq \frac{1}{3}(a+b+c)^2-\frac{4}{3}(a+b+c)+1 =\frac{1}{3}(a+b+c-1)(a+b+c-3)\geq 0$

(vì $a+b+c \geq 3$)

thánh đọc đề  :icon6:  :icon6:




#684634 CMR: $a^2+b^2+c^2-4abc+1\geq 0$

Posted by Uchiha sisui on 15-06-2017 - 15:28 in Bất đẳng thức - Cực trị

Bài này đơn giản thôi  :icon6:  :icon6:

 

Ta có Dễ chứng minh $\sum \frac{1}{a}\geq 3$ $\Rightarrow  ab+bc+ca\geq 3abc$  (*)

 

Theo bổ đề quen thuộc: $\sum a^{2} +2abc+1\geq 2(ab+bc+ca)$

 

Bổ đề trên đã quá quen thuộc xin phép không chứng minh, có thể sử dụng Schur kết hợp AM-GM.

 

Sử dụng Bổ đề ta có: Bất đẳng thức cần chứng minh tương đương với $\sum a^{2} +2abc+1\geq 6abc$

 

Ta sẽ chứng minh $2(ab+bc+ca)\geq 6abc\Leftrightarrow ab+bc+ca\geq 3abc$. Bất đẳng thức cuối luôn đúng theo (*). Ta có điều phải chứng minh. Dấu bằng xảy ra khi và chỉ khi $a=b=c=1$ :closedeyes:  :closedeyes:  :closedeyes:




#684691 CMR: $a^2+b^2+c^2+a+b+c\geq \frac{18}{a+b+c...

Posted by Uchiha sisui on 16-06-2017 - 14:58 in Bất đẳng thức - Cực trị

Bản chất vẫn là Schur cái PQR ghi lại cho đẹp mắt thôi :)




#691514 USAMO 2017 ngày 1

Posted by Uchiha sisui on 25-08-2017 - 16:03 in Thi HSG Quốc gia và Quốc tế

Bài 3.

 

Đoạn đầu có thể xử lý được bằng trực đẳng phương như sau:

 

Gọi $W,G$ lần lượt là trung điểm của $BC$, cung $BC$ chứa $A$.

 

Xét ba đường tròn $(DM)$, $(\varpi )$, $(ADWG)$ thì ta có trục đẳng phương của ba đường tròn đồng quy tại $S$

 

Do đó dễ chứng minh $SA$ là phân giác ngoài của tam giác ABC tại đỉnh A.

 

Còn đoạn sau mình xử lý giống bạn dogsteven.




#681150 Đề thi thử THPT chuyên KHTN Lớp 9 Vòng 2 - Đợt 4 Năm 2017

Posted by Uchiha sisui on 19-05-2017 - 11:41 in Tài liệu - Đề thi

Còn câu nữa xử nốt cho nó lành

Câu II 

2)Gọi $X_{n}$ là số cách sắp xếp n người ($n\geq 0$) thỏa mãn đề bài (coi như $X_{0}=1$ vì khi không có người nào đứng thì chỉ có một cách sắp xếp duy nhất :) )

Giả sử chiều cao của $n$ người trên lần lượt là $a_{1};a_{2};...;a_{n}$ sao cho với $i>j$ thì $a_{i}>a_{j}$

Khi đó; giả sử người có chiều cao là $a_{1}$ đứng ở vị trí thứ $k$($0\leq k\leq n$) thì mỗi người đứng ở vị trí từ $1\rightarrow k$ luôn cao hơn tất cả những người đứng trước vì nếu ngược lại thì tồn tại một người có chiều cao nhỏ hơn $a_{1}$ (vô lý) $\Rightarrow$ người đứng ở vị trí thứ 1 là người cao nhất và có chiều cao $a_{n}$

Tương tự; người đứng ở vị trí thứ 2 có chiều cao là $a_{n-1}$

                 người đứng ở vị trí thứ 3 có chiều cao là $a_{n-2}$

                 .............

                 người đứng ở vị trí thứ $k-1$ có chiều cao là $a_{n-k}$

Do đó với mọi $0\leq k\leq n$ thì chỉ có một cách sắp sếp những người đứng ở vị trí từ 1 đến k sao cho thỏa mãn đề bài. Từ đó suy ra số cách sắp xếp n người thỏa mãn đề bài khi người thấp nhất (có chiều cao $a_{1}$) đứng ở vị trí thứ $k$ chính bằng số cách sắp xếp $n-k$ người đứng từ vị trí thứ $k+1$ trở đi ($=X_{n-k}$).

Vì vậy; khi cho $k$ lần lượt bằng $n;n-1;...;1$ thì ta nhận được số cách sắp xếp $n$ người lần lượt là $X_{0};X_{1};X_{2};...;X_{n-1}$

Mà tổng số cách sắp xếp $n$ người bằng tổng các số trên nên $X_{n}=X_{0}+X_{1}+...+X_{n-1}$

Lại có $X_{0}=1$ và $X_{1}=1$ nên từ công thức trên và bằng phương pháp quy nạp ta chứng minh được $X_{n}=2^{n-1} (n\geq 1)$

Thay $n= 10$ vào trên ta có $X_{10}=512$

P/S: Mới off có sáng với chiều chủ nhật để đi chơi mà tối lên diễn đàn đã thấy đề KHTN bị "xơi" gần hết rồi; còn mỗi cấu tổ :))

bác giỏi ghê :v trâu quá :V




#684638 $\frac{1}{x}+\frac{1}{y...

Posted by Uchiha sisui on 15-06-2017 - 15:47 in Bất đẳng thức - Cực trị

Bài này đơn giản thôi '' xử đẹp'' bằng PQR  :closedeyes:  :closedeyes:

 

Lời giải

 

Sử dụng BĐT quen thuộc ta dễ chứng minh $1\geq 3(xy+yz+zx)\Rightarrow 1\geq 3q$

 

Với $p=x+y+z=1, q=xy+yz+zx, r=xyz$

 

Ta sử dụng bổ đề sau: $r\leq \frac{q^{2}(7-16q)}{5(5-12q)}$. Đây là bổ đề quen thuộc nên tôi không chứng minh lại  :closedeyes:  :closedeyes:

 

Bất đẳng thức cần chứng minh tương đương với:

 

$\frac{q}{r}+48q-25\geq \frac{5(5-12q)}{q(7-16q)}+48q-25=\frac{(1-3q)(5-16q)^{2}}{q(7-16q)}\geq 0$

 

 




#693767 Đề chọn đội tuyển QG Dak Lak năm 2017-2018

Posted by Uchiha sisui on 26-09-2017 - 22:30 in Thi HSG cấp Tỉnh, Thành phố. Olympic 30-4. Đề thi và kiểm tra đội tuyển các cấp.

Bài hình sử dụng hoàn toàn kiến thức lớp 9 và nó là 1 bài toán quen thuộc của lớp 9!